Which of the following is not a congruence theorem or postulate?
A.) AAS
B.) SSS
C.) AA
D.) SAS

Answers

Answer 1

Answer:

C.) AA

Step-by-step explanation:

AA is a similarity theorem

hope this helps stay safe :)

Answer 2

Answer:

The answer would be C.

Step-by-step explanation: Hope this helps :)


Related Questions

(c) 0.34 L = _________________ ml ​

Answers

Answer:

340 ml

Step-by-step explanation:

What percentage of temperatures are below 55°? A. 50% B. 25% C. 75% D. 20%

Answers

Step-by-step explanation:

C

im not sure of my answer. hope to help

I think d because others one

Find the intersection point between the lines of equations:

2x-y+6=0 and 2x+3y-6=0 ​

Answers

Step-by-step explanation:

The two equation will intersect each other at the point which will be the solution of the given two equations , and the given equations are ,

[tex]\implies 2x -y +6=0\\\\\implies 2x + 3y -6=0[/tex]

On subtracting the given equations we have,

[tex]\implies -y - 3y +6 -(-6) = 0 \\\\\implies -4y = -12 \\\\\implies y = -12/-4\\\\\implies y = 3 [/tex]

Put this value in any equation , we have ,

[tex]\implies 2x -3 +6 =0\\\\\implies 2x = -3 \\\\\implies x =\dfrac{-3}{2} \\\\\implies x =-1.5 [/tex]

Hence the lines will Intersect at ,

[tex]\implies\underline{\underline{ Point=(-1.5, 3)}}[/tex]

for the first one x = 1/2 y-3" and y = 2 x + 6 and for the other one is x =  − 3 /2  y+ 3  and y= − 2 /3  x + 2

how i did this Step 1: Add -3y to both sides.

Step 2: Add 6 to both sides.

Step 3: Divide both sides by 2.

The question is in the photo

Answers

Answer:

Step-by-step explanation:

This is a right triangle because angle V is 90. Draw out a right triangle and put V at the 90 degree angle. It doesn't matter where you put U or T; you get the same cos value for T regardless of how you place your other 2 angles. The things you need to know are that UT is the hypotenuse of the triangle, side VU is across from angle T, and side TV is across from angle U.

The cos ratio is side adjacent to the reference angle over the hypotenuse.

Our reference angle is T, so we are looking for the side next to T that is NOT the hypotenuse. This side measures 33; the hypotenuse measures 65, so the tangent ratio of T is

[tex]tanT=\frac{33}{65}[/tex]

A cylinder has a volume of 245x cubic units and a helght of 5 units. The diameter of the cylinder is
7 units
14 units
49 units

Answers

Answer:

Diameter = 14 units

Step-by-step explanation:

Volume ofa cylinder = πr²h

Volume of the cylinder = 245π cubic units

Height = 5 units

Volume of a cylinder = πr²h

245π = π × r² × 5

245π = 5r²π

Divide both sides by π

245π / π = 5r²π / π

245 = 5r²

r² = 245/5

= 49

r² = 49

r = √49

r = 7 units

Diameter = 2 × radius

= 2 × 7 units

= 14 units

Diameter = 14 units

Find the measure of ∠AED for m∠BEC = 118

Answers

Answer: 118°

Step-by-step explanation:

If px-3+qx+1=2x+3,Find the value of p+q

Answers

Answer: p + q = (2x+5)/x

isolate p + q on one side: therefore rewrite the equation.

px + qx -2 = 2x + 3

px + qx = 2x + 5

x(p+q) = 2x+5

p + q = (2x+5)/x

PLS HELP !!!
a. 15
b. 1
c. 2
d. 11

Answers

Answer:

d

Step-by-step explanation:

ive had this exac problem before

Answer:

B. 1

Step-by-step explanation:

Since the sides are equal to each other, the equations will be equal.

4x + 7 = x + 10

      -7          -7

---------------------

4x= x + 3

-x    -x

-------------

3x = 3

----   ---

3      3

x = 1

The answer is 1.

Dog breeds 2238 pages in seven hours he reads the same number of pages each hour how many pages did he read in one hour

Answers

Answer:

319 pages and a half

Step-by-step explanation:

Divide 2238 by seven. The answer is about 319.71.

Can someone help me with this math homework please!

Answers

Answer:

the correct options are 1, 2, 5, and 6th

1. and 2.

all functions have a dependant variable y and independent variable x, and the set of dependant variables is called its range whereas the set of independent variables is called its domain.

5. and 6.

In a horizontal line 1 input associates itself with exactly one ouput, even tho the output is constant (same) for all values of x, and that's the property of a function

Answer:

all functions have dependent variables

all functions have an independent variable

a horizontal line Is an example of a functional relationship

help pls!!

The functionſ is defined by f(x) = x(x + 3). If f(a) = 40 and a > 0, what is the value of a ?
A) 3
B) 5
C) 7
D) 8

Answers

Answer:

B) 5

Step-by-step explanation:

We are given the function:

[tex]f(x)=x(x+3)[/tex]

We are given that f(a) = 40 and a > 0 and we want to determine the value of a.

Substitute:

[tex]f(a)=40=a(a+3)[/tex]

Distribute:

[tex]a^2+3a=40[/tex]

Subtract 40 from both sides:

[tex]a^2+3a-40=0[/tex]

We can factor using 8 and -5. Hence:

[tex](a+8)(a-5)=0[/tex]

By the Zero Product Property:

[tex]a+8=0\text{ or } a-5=0[/tex]

Solve for each case:

[tex]\displaystyle a=-8\text{ or } a=5[/tex]

Since a > 0, we can eliminate the first solution. Hence:

[tex]a=5[/tex]

Our answer is B.

What number is increased by 40% become 28?
a.5 b. 10 c. 15 d. 20​

Answers

Answer:

d. 20

Step-by-step explanation:

28 ÷ (100%+40%)

= 28 ÷ 140%

= 28 ÷ 1.4 = 20

Answer:

option d

Step-by-step explanation:

Let the number be x

40% of x = 0.40x

Increased by 40% = 28

That is ,

           x + 0.40x = 28

               1.40x = 28

                   [tex]x = \frac{28}{1.40}\\[/tex]

                  [tex]x = \frac{28 \times 100}{1.40 \times 100 } = \frac{28 \times 100}{140} =\frac{4 \times 100}{20} = 4 \times 5 = 20[/tex]

Therefore, the number is 20

Calculate the average rate of change of a function over a specified interval. Which expression can be used to determine the average rate of change in f(x) over the interval 2, 9?

Answers

Given:

The interval is [2,9].

To find:

The average rate of change in f(x) over the interval [2,9].

Solution:

The average rate of change in f(x) over the interval [a,b] is defined as:

[tex]m=\dfrac{f(b)-f(a)}{b-a}[/tex]

In the interval [2,9], the value of a is 2 and the value of b is 9.

Using the above formula, the average rate of change in f(x) over the interval [2,9] is:

[tex]m=\dfrac{f(9)-f(2)}{9-2}[/tex]

[tex]m=\dfrac{f(9)-f(2)}{7}[/tex]

Therefore, the required expression for the average rate of change in f(x) over the interval [2,9] is [tex]\dfrac{f(9)-f(2)}{9-2}[/tex], it is also written is [tex]\dfrac{f(9)-f(2)}{7}[/tex].

Answer:

D

Step-by-step explanation:

right on edge

nth term of an AP is given by an expression 7n-2,find the common difference of this sequence if fisrt term of sequence is 1? a.2 b.-2 c-7 d7​

Answers

Step-by-step explanation:

there is something wrong with what your write here as problem description.

either there is something missing in the main expression, or in the answer options.

given your nth-term expression 7n-2, none of the 4 answer options can fit.

so, I thought, maybe there was a typo.

the possible interpretations of the expression could be

7n - 2

7(n-2)

[tex] {7}^{n} - 2[/tex]

[tex] {7}^{n - 2} [/tex]

we know that

a1 = 1

a2 = either

a1 + 2 = 3

a1 - 2 = -1

a1 + 7 = 8

a1 - 7 = -6

so, we can eliminate both negative answer options, because they would cause only negative sequence elements, but the main expression (neither of the 4 possibilities) does not allow that.

but also none of the 4 possibilities of the expression delivers any of the 4 values for a2 as described above.

7×2 - 2 = 14 - 2 = 12

7(2-2) = 7×0 = 0

7² - 2 = 49 - 2 = 47

[tex] {7}^{2 - 2} = {7}^{0} = 1[/tex]

similar not for a3.

so, if I consider your answer options right, we have 4 possible arithmetic sequences:

1. 1, 3, 5, 7, 9, 11, 13, 15, ...

2. 1, -1, -3, -5, -7, -9, -11, -13, ...

3. 1, 8, 15, 22, 29, 36, 43, 50, ...

4. 1, -6, -13, -20, -27, -34, -41, ...

the nth term expression is for

a. an = 2n - 1

b. an = 2×(-n) + 3 = 2×(-n + 1) + 1

c. an = 7n - 6

d. an = 7×(-n) + 8 = 7×(-n + 1) + 1

so, please pick the right expression from this list, and then use the answer option with the same letter.

the regular price of base ball cleats is 80 dollars if the cleats are on sale for 45% off, then what is the value of the discount in dollars?

Answers

Answer:

$36

Step-by-step explanation:

If the cleats were on sale for 45% off, that means they were 55% "on." Therefore, you have to multiply 80 by 0.55* to get how much they cost.

80 x 0.55 is 44.

To get the value of the discount, you subtract 44 from 80, which is 36.

The cleats sold for $44, and there was a $36 discount.

*you multiply by 0.55 because it is 55 percent. Percent means "per every hundred." That's why you have to divide 55 by 100 to get 0.55.

An alternate way to solve this is to multiply 80 by 55 and then divide the result by 100, but you'll still get 44.

If f(x)=3x^(2)+1 and G(x)=2x-3 what would f(f(x))

Answers

Answer:

f(f(x)) = 27[tex]x^{4}[/tex] + 18x² + 4

Step-by-step explanation:

To find f(f(x)) substitute x = f(x) into f(x) , that is

f(3x² + 1)

= 3(3x² + 1)² + 1 ← expand parenthesis using FOIL

= 3(9[tex]x^{4}[/tex] + 6x² + 1) + 1 ← distribute parenthesis by 3

= 27[tex]x^{4}[/tex] + 18x² + 3 + 1 ← collect like terms

= 27[tex]x^{4}[/tex] + 18x² + 4

Hello,

[tex](fof)(x)=f(f(x))\\\\=3(3x^2+1)^2+1\\\\=3(9x^4+6x^2+1)+1\\\\\boxed{=27x^4+18x^2+4}[/tex]

what is the area of this triangle​

Answers

Answer:

son Judy even eat and technology and yyry and then he will get your BMW

Find the missing angle in the image below. Do not include spaces in your answers ** Can somebody help me fr everybody keep giving me the wrong answer

Answers

Answer:

measure of angle V + measure angle W = VUF

Answer:

∠ VUF = 94°

Step-by-step explanation:

The exterior angle of a triangle is equal to the sum of the 2 opposite interior angles

∠ VUF is an exterior angle of the triangle, then

∠ VUF = 71° + 23° = 94°

Solve for x. Round to the nearest tenth, if necessary. Calculating Sin or Cos or Tan

Answers

Answer:

1.06

Step-by-step explanation:

cos41⁰ = 0.8 ÷ X

X = 0.8 ÷ cos41⁰

Answer:

Step-by-step explanation:

Use SOH CAH TOA to recall how the trig functions fit on a right triangle

SOH: Sin(Ф)= Opp / Hyp

CAH: Cos(Ф)= Adj / Hyp

TOA: Tan(Ф) = Opp / Adj

x = Hyp  (for this question)

Cos(41) = 0.8 / Hyp

use your algebra skillz

Hyp = 0.8 / Cos(41)

use your calculator to fund Cos(41) into 0.8

Hyp = 1.060010395  (that's all the decimal places my calc would go out to )

x = 1.1  ( rounded to nearest 10th )

A committee that consists of five members are to be chosen from 6 boys and 5 girls. Find the number of different committees that can be formed if only two boys are selected​

Answers

Answer:

150 different committees can be formed

Step-by-step explanation:

We have 6 boys and 5 girls and we want to select 5 members

Out of these 5 members, two boys are selected

Since two boys are selected, we are left with three girls

So, out of 6 boys, we select 2 boys and out of 5 girls, we select 3 girls

Mathematically, we know that the number of ways in which we can select r items from a total n follows the combinatorial formula;

nCr = n!/(n-r)!r!

With this, we have;

6C2 * 5C3

= (6!/(6-2)!2!) * (5!/(5-3)!3!) = 150 different committees can be formed

Which fraction is the largest? 7/9 3/4 1/2 2/3

Answers

Rewrite the fractions as decimals by dividing:

7/8 = 0.7777

3/4 = 0.75

1/2 = 0.5

2/3 = 0.666

0.777 is the largest number so 7/9 is the largest fraction.

Answer: 7/9

Answer:

2/3

Step-by-step explanation:

L.C.M:36

7/9:28/36

3/4:27/36

1/2:1836

2/3:36/36

Triangle Angle-Sum Theorem

Answers

Answer:

answer is C , 123

Step-by-step explanation:

angle 1 = sum of the interior opposite Angles

1 = 60 + 63

1 = 123

The 20th term of an arithmetic progression 1,5,9,13......​

Answers

Answer:

a₂₀ = 77

Step-by-step explanation:

There is a common difference between consecutive terms , that is

5 - 1 = 9 - 5 = 13 - 9 = 4

This indicates the sequence is arithmetic with nth term

[tex]a_{n}[/tex] = a₁ + (n - 1)d

where a₁ is the first term and d the common difference

Here a₁ = 1 and d = 4 , then

a₂₀ = 1 + (19 × 4) = 1 + 76 = 77

How many possible outcomes do you have when you role 4 dice

Answers

Answer:

1296

Step-by-step explanation:

6^4

use the substitution method to solve the system of equations

3x+2y=13

y=x-1

Answers

Answer:

[tex]x=3\\y=2[/tex]

Step-by-step explanation:

[tex]3x+2y=13\\y=x-1[/tex]

Solve for [tex]y=x-1[/tex] for y:

[tex]y=x-1[/tex]

Substitute [tex]x-1[/tex] for y in [tex]3x+2y=13[/tex]:

[tex]3x+2y=13[/tex]

[tex]3x+2(x-1)=13\\[/tex]

[tex]5x-2=13[/tex]

[tex]5x=13+2[/tex]

[tex]5x=15[/tex]

[tex]x=15/5[/tex]

[tex]x=3[/tex]

Substitute 3 for x in [tex]y=x-1[/tex]:

[tex]y=x-1[/tex]

[tex]y=(3)-1[/tex]

[tex]y=2[/tex]

[tex]x=3[/tex] and [tex]y=2[/tex]

hope this helps...

please say me the answer fast with step by step process​

Answers

Yes I’m sorry but I’m sorry I’m texting back to my sister in

Which segment is adjacent to∠N?

Answers

Answer:

SN

Step-by-step explanation:

The line segment that is adjacent to angle ∠N will be NS and NJ. Then the correct options are A and B.

What is the triangle?

The polygonal shape of a triangle has a number of sides and three independent variables. Angles in the triangle add up to 180°.

A line segment in mathematics has two different points on it that define its boundaries. A line segment is sometimes referred to as a section of a path that joins two places.

The triangle ΔNSJ is a right angle triangle in which the angle ∠NSJ will be a right angle that is 90°.

The line segment that is adjacent to angle ∠N will be NS and NJ. Then the correct options are A and B.

More about the triangle link is given below.

https://brainly.com/question/25813512

#SPJ2

At a school concert the total value of tickets sold was $4452. Student tickets sold for $8 and adult tickets sold for $12. The number of adult tickets sold was 7 less than 4 times the number of student tickets. Find the number of student tickets sold.

Answers

Answer:

there is 81 student and 317 adults adults :) got u

Step-by-step explanation:

4452 = 8n + (4n-7)12

4452 = 8n + 48n - 84

4536 = 56n

81 = n

FOR EASY BRAINLIEST ANSWER QUESTION BELOW!

1. Solve each word problem .twice a number added three times the sum of the number and 2 is more than 17. Find the numbers that satisfy condition

Answers

Answer:

56

Step-by-step explanation:

write a equation for y=|x| if the graph is translated right by 3 units and down by 1 unit

Answers

Answer:

y=|x -3| -1

Step-by-step explanation:

Other Questions
A 25.0kg girl pushes a 50.0kg boy with a force of 100.0N. What is the acceleration of the girl? Which sentence contains an infinitive?Help pls Which one from the passage best symbolizes Magis character 5 plus 6 times 8 plus 9 times 10 plus 8 plus 4 plus 2 plus 9 plus 8 plus 7 plus 5 plus 7 plus 6 times 7 times 9 times 8 times 5 times 4 times 3 times 2 times 1 times 6 times 8 times 9 times 12 times 17 times 19 times 20 times 12 times 11 times 13 times 14 times 15 times 16 The volume of the sphere is500-n cubic units.3What is the value of x?O4 unitsO 5 unitsO 8 unitsO 10 units QUESTIONS2. When you are sharing the road with a bicyclist or motorcyclist, you should:A. Always allow them to share your laneB. Expect them to yield the right-of-way to carsC. Treat them as you would treat any driver What is the correct spelling for: Go to the hardware store and pick up the following items; A saw, A hammer And some rope. Read the passage below and answer the question."...and in the fourteenth [fifteenth] century the whole English army, after ravaging the kingdom of France, was driven back like men petrified with fear; and this brave exploit was performed by a few broken forces collected and headed by a woman, Joan of Arc."Why does Paine most likely refer to Joan of Arc?A. to enlist the aid of the French government in the fight against BritainB. to show that the Americans have God on their side just as Joan of Arc didC. to demonstrate how cowardly the British army can beD. to prove that the British army had been defeated in the past If BcA, AnB=(1,4,5)and AuB= (1,2,3,4,5,6) find B? Tamir wants to buy a snowboard. The original price is $760. How much will Tamir pay if he buys it during the sale? A satellite of mass 5460 kg orbits the Earth and has a period of 6520 sA)Determine the radius of its circular orbit.B)Determine the magnitude of the Earth's gravitational force on the satellite.C)Determine the altitude of the satellite. You just returned from some extensive traveling.You started your trip with $10,000 in your pocket.You spent 1.32 million pesos in Chile where Ps1 = $.001642.You spent Ps36,000 in Uruguay where Ps1 = $.03526.Then on the way home,you spent Ps29,000 in Mexico where $1 = Ps18.8709.How many dollars did you have left by the time you returned to the U.S.? A) $3,889.07 B) $4,001.84 C) $4,110.27 D) $5,026.44 E) $4,299.03 A body initially at rest travels a distance 100 m in 5 s with a constant acceleration. calculate(i) Acceleration(ii) Final velocity at the end of 5 s. An electric eel can generate a 180-V, 0.1-A shock for stunning its prey. What is the eel's power output The chemical formula is different from the empirical formula in Factorise this equasionX^2-5 what affect did ice age have on the planet Observe as equaes e de acordo com Brnsted-Lowry, os compostos destacados so, respectivamente Although many steps have already been taken, the restructuring is ________ considered to be in its early phases.Select one:A. stillB. alreadyC. yetD. rarely It's time to buy pet food again and Lisa heads to the grocery store with $40 in her purse, leaving her four hungry dogs and seven hungry cats at home. Dog food costs $1 per can and cat food costs $0.50 per can. Lisa wants to minimize her pet food cost. What is an appropriate objective function for this scenario?